信号与系统dsl考题

信号与系统dsl考题

ID:42093180

大小:196.52 KB

页数:9页

时间:2019-09-07

信号与系统dsl考题_第1页
信号与系统dsl考题_第2页
信号与系统dsl考题_第3页
信号与系统dsl考题_第4页
信号与系统dsl考题_第5页
资源描述:

《信号与系统dsl考题》由会员上传分享,免费在线阅读,更多相关内容在工程资料-天天文库

1、一、填空题(30分,每题4分,第6小题每空3分)1・已知f(t)=(t2+3)u(t),求f"(t)=;2.信号通过系统的不失真传输条件是该系统函数满足H(j3)=;3.已知一个系统的输入输出关系为y(t)=f(3t)+3,则该系统是系统;4・[(f+2t+l)8(t+l)dt=;5.下列信号中,是周期信号;a)x(n)=2cos(3n+n/4)b)x(t)=e2tsin(2t)c)x(t)=sin(2nt)g(t)*d)x(n)二E[§(n-3m)-8(n-3m-l)]6・若信号通过LTI系统后零状态响应为y(t)=Kf(K,t0为常数),贝9该系统频率特

2、性H(jo)=,单位冲激响应h(t)=;7・已知信号x(t)=eatu(t),a>0,则该信号的拉普拉斯变换及收敛域为。二、已知信号fo(t)的傅立叶变换为F°(w),求下列信号的傅立叶变换。(15分)(1)f(t)=f0(t)*6(t-nT)(2)f(t)=(t)*5(t-nT)三、已知差分方程y(n)+0.2y(n・l)・0.24y(n-2)=x(n)+x(n-1)(10分)(1)求相应系统传递函数H(z)(8分)(2)设系统为因果系统,根据H(z)确定其稳定性。(7分)四、已知某信号f(t)最高频率为其傅立叶频谱函数为F(s),将该信号与周期为T,脉宽

3、为t,幅度为1的周期矩形脉冲信号相乘,求乘积信号的频谱函数。(15分)五、设某LTI系统单位阶跃响应为2e2tu(t)+6(t),计算系统对于信号eAi(t)的输出信号y(t)。(15分)六、某离散系统如图:(20分)=2.08o七、计算卷积e_3tu(t-l)*e_3tu(t+3)o(10分)答案:、1・2u(t)+4t5(t)+(t2+3)55(t)2.Ke_jGJt03.非线性4.05.D6.K严;K5(t-to)7.l/(s-a);o>a二、(1)因为f(t)=f0(t)*5(t-nT)=f0(t-nT),因此根据傅立叶变换时移特性得:F(o)=F0

4、(o)gj^nT(2)f(t)=^fo(t)*5(t-nT)=J^fo(t-nT)因此信号为周期信号,根据周期信号频谱与非周期信号频谱关系可得:F(3)=F0[3]三§(3・n3])],其中3]=2n/T三、解:(1)对差分方程两边作z变换有:Y⑵+0.2z_1Y(z)-0.24z-2Y(z)=X(z)+z_1X(z)H(z)=Y(z)/X(z)=(l+z'1)/(l+0.2z1-0.24z2)(3)对传递函数H⑵因式分解有:H(z)=(l+z1)/(1・0.4z")(1-0.6z_1)由于系统为因果系统,所以H⑵的收敛域为lzl>0.6,显然单位圆被包含在

5、收敛域内,因此答案:、1・2u(t)+4t5(t)+(t2+3)55(t)2.Ke_jGJt03.非线性4.05.D6.K严;K5(t-to)7.l/(s-a);o>a二、(1)因为f(t)=f0(t)*5(t-nT)=f0(t-nT),因此根据傅立叶变换时移特性得:F(o)=F0(o)gj^nT(2)f(t)=^fo(t)*5(t-nT)=J^fo(t-nT)因此信号为周期信号,根据周期信号频谱与非周期信号频谱关系可得:F(3)=F0[3]三§(3・n3])],其中3]=2n/T三、解:(1)对差分方程两边作z变换有:Y⑵+0.2z_1Y(z)-0.24z

6、-2Y(z)=X(z)+z_1X(z)H(z)=Y(z)/X(z)=(l+z'1)/(l+0.2z1-0.24z2)(3)对传递函数H⑵因式分解有:H(z)=(l+z1)/(1・0.4z")(1-0.6z_1)由于系统为因果系统,所以H⑵的收敛域为lzl>0.6,显然单位圆被包含在收敛域内,因此系统为稳定系统。四、解:根据傅立叶变换特性,时域相乘即频域卷积,再利用周期信号频谱特点,可以直接得到:F2(3)=(l/2n)F(3)*{TSa(3t/2)・[8(co-n«0)]其中卷积的第一项是带阻信号的频谱,而第二项是周期矩形脉冲信号的频谱,该项可以利用周期信号

7、频谱与非周期信号频谱Z间的关系得:F2()=(t/T)-?-Sa(nco0t/2)F(co-nco0)五、解:令阶跃响应g(t)=2八”⑴+犯),则系统的冲击响应:h(t)=—g(r)=+2J(r)-4e~2tu(t)dt则L[y(t)]=Y(s)二厶[3宀(r)]・咖)]丄+—丄旦S+1$+15+15+2=3-125+2所以,输出信号:y(t)=L[Y(s)]=35a)-[9厂-12严]•“⑴六、解:(1)由离散系统框图得出:y(n)=x(n)+x(n-1)-0.2y(n-1)4-0.24y(n—2)整理得y(n)+0.2y(n-1)-0.24y(n-2)

8、=x(n)+x(n一1)(1)对上式两边取单边z变换

当前文档最多预览五页,下载文档查看全文

此文档下载收益归作者所有

当前文档最多预览五页,下载文档查看全文
温馨提示:
1. 部分包含数学公式或PPT动画的文件,查看预览时可能会显示错乱或异常,文件下载后无此问题,请放心下载。
2. 本文档由用户上传,版权归属用户,天天文库负责整理代发布。如果您对本文档版权有争议请及时联系客服。
3. 下载前请仔细阅读文档内容,确认文档内容符合您的需求后进行下载,若出现内容与标题不符可向本站投诉处理。
4. 下载文档时可能由于网络波动等原因无法下载或下载错误,付费完成后未能成功下载的用户请联系客服处理。